Proof of a Limit Law: If lim f(x) = L, lim g(x) = M, then...

o_O

Full Member
Joined
Oct 20, 2007
Messages
396
Question: If \(\displaystyle \lim_{x \to a} f(x) = L\) and \(\displaystyle \lim_{x \to a} g(x) = M\), then \(\displaystyle \lim_{x \to a} (f(x)g(x)) = LM\).

Proof from James Stewarts':

Let \(\displaystyle \epsilon > 0\) . We want to find \(\displaystyle \delta > 0\) such that \(\displaystyle |f(x)g(x) - LM|\) whenever \(\displaystyle 0 < |x - a| < \delta\).

\(\displaystyle \left| f(x)g(x) - LM \right|\)

\(\displaystyle = \left| f(x)g(x) - Lg(x) + Lg(x) - LM \right|\)

\(\displaystyle = \left|\left[f(x) - L\right]g(x) + L\left[g(x) - M\right]\right|\)

\(\displaystyle \leq \left|\left[f(x) - L\right]\left(g(x)\right)\right| + \left|\left(L\right)\left[g(x) - M\right]\right|\) (Triangle inequality)

\(\displaystyle = \left|f(x) - L\right|\left|g(x)\right| + \left|L\right|\left|g(x) - M\right|\)

We want to make each of these terms less than \(\displaystyle \frac{\epsilon}{2}\).

Since \(\displaystyle \lim_{x \to a} g(x) = M\), there is a number \(\displaystyle \delta_{1} > 0\) such that: \(\displaystyle |g(x) - M| < \frac{\epsilon}{2\left(1 + |L|\right)}\)* whenever \(\displaystyle 0 < |x - a| < \delta_{1}\).

Also, there is \(\displaystyle \delta_{2}>0\) such that if \(\displaystyle 0 < |x - a| < \delta_{2}\), then \(\displaystyle |g(x) - M| < 1\) and therefore:

\(\displaystyle \left|g(x)\right| = \left|g(x) - M + M\right| \hspace{4mm} \leq \hspace{4mm} \left|g(x) - M\right| + \left|M\right| \hspace{4mm}< \hspace{4mm} 1 + \left|M\right|\)

Since \(\displaystyle \lim_{x \to a}f(x) = L\), there is a number \(\displaystyle \delta_{3}>0\) such that: \(\displaystyle \left|f(x) - L\right|<\frac{\epsilon}{2\left(1+|M|\right)}\)* whenever \(\displaystyle 0 < |x - a| < \delta_{3}\).

Let \(\displaystyle \delta = min\left\{\delta_{1},\delta_{2},\delta_{3}\right\}\). If \(\displaystyle 0 < |x - a| < \delta\).

Then we have \(\displaystyle 0 < |x-a| <\delta_{1}, \hspace{4mm} 0 < | x-a| < \delta_{2}, \hspace{4mm} 0 <|x-a|<\delta_{3}\)

so we can combine the inequalities to obtain:

\(\displaystyle \left|f(x)g(x) - LM\right|\)

\(\displaystyle \leq \hspace{4mm}\left|f(x)-L\right| \left|g(x)\right| \hspace{4mm}+ \hspace{4mm}\left|L\right|\left|g(x)-M\right|\)

\(\displaystyle = \frac{\epsilon}{2\left(1 + |M|\right)} \left(1 + |M|\right) \hspace{4mm} + \hspace{4mm} |L| \frac{\epsilon}{2\left(1 + |L|\right)\)

\(\displaystyle < \frac{\epsilon}{2} + \frac{\epsilon}{2} = \epsilon\)

Therefore, \(\displaystyle \lim_{x \to a} f(x)g(x) = LM\)
Problem: I don't exactly how they made the statements with the *s. Where did these inequalities come from and how can they be asserted? Also, I'm kind of iffy on why we need three \(\displaystyle \delta\)s ... Any help would be appreciated!
 
Given the extent and fundamental nature of these questions, I think that you should sit down with a live tutor. These are important concepts to grasp.
 
Top